1
$\begingroup$

The formula: $\lim_{n \to \infty}\dfrac{\log (1 - n + n^2)}{\log (1 + n + n^{10})^{1/3}}.$

Thanks for any advice!

  • 1
    For some basic information about writing math at this site see e.g. [here](http://meta.math.stackexchange.com/questions/5020/), [here](http://meta.stackexchange.com/a/70559/155238), [here](http://meta.math.stackexchange.com/questions/1773/) and [here](http://math.stackexchange.com/editing-help#latex).2012-10-17

2 Answers 2

3

Hint:

\begin{eqnarray*} \frac{\log \left( 1-n+n^{2}\right) }{\log \left( 1+n+n^{10}\right) ^{1/3}} &=&\frac{\log \left( 1-n+n^{2}\right) }{\frac{1}{3} \log \left( 1+n+n^{10}\right) } \\ &=&\frac{\log \left( n^{2}\left( 1/n^{2}-1/n+1\right) \right) }{\frac{1}{3} \log \left( n^{10}\left( 1/n^{10}+1/n^{9}+1\right) \right) } \\ &=&\frac{\log n^{2}+\log \left( 1/n^{2}-1/n+1\right) }{\frac{1}{3}\log n^{10}+\frac{1}{3}\log \left( 1/n^{10}+1/n^{9}+1\right) }. \end{eqnarray*}

ADDED. Now that there is already a full answer I complete mine. Using the rule $\log a^{r}=r\log a$, as commented by David Mitra, and $\log ab=\log a+\log b$, manipulate algebrically the fraction and rewrite it as \begin{eqnarray*} \frac{\log \left( 1-n+n^{2}\right) }{\log \left( 1+n+n^{10}\right) ^{1/3}} &=&\frac{\log \left( n^{2}\left( 1/n^{2}-1/n+1\right) \right) }{\frac{1}{3}% \log \left( 1+n+n^{10}\right) } \\ &=&\frac{\log \left( n^{2}\left( 1/n^{2}-1/n+1\right) \right) }{\frac{1}{3}% \log \left( n^{10}\left( 1/n^{10}+1/n^{9}+1\right) \right) } \\ &=&\frac{\log n^{2}+\log \left( 1/n^{2}-1/n+1\right) }{\frac{1}{3}\log n^{10}+\frac{1}{3}\log \left( 1/n^{10}+1/n^{9}+1\right) } \\ &=&\frac{2\log n+\log \left( 1/n^{2}-1/n+1\right) }{\frac{1}{3}\times 10\log n+\frac{1}{3}\log \left( 1/n^{10}+1/n^{9}+1\right) } \\ &=&\frac{2+\frac{\log \left( 1/n^{2}-1/n+1\right) }{\log n}}{\frac{10}{3}+% \frac{1}{3}\frac{\log \left( 1/n^{10}+1/n^{9}+1\right) }{\log n}}. \end{eqnarray*}

We thus have \begin{eqnarray*} \lim_{n\rightarrow \infty }\frac{\log \left( 1-n+n^{2}\right) }{\log \left( 1+n+n^{10}\right) ^{1/3}} &=&\lim_{n\rightarrow \infty }\frac{2+\frac{\log \left( 1/n^{2}-1/n+1\right) }{\log n}}{\frac{10}{3}+\frac{1}{3}\frac{\log \left( 1/n^{10}+1/n^{9}+1\right) }{\log n}} \\ &=&\frac{2+\displaystyle\lim_{n\rightarrow \infty }\frac{\log \left( 1/n^{2}-1/n+1\right) }{\log n}}{\frac{10}{3}+\frac{1}{3}\displaystyle% \lim_{n\rightarrow \infty }\frac{\log \left( 1/n^{10}+1/n^{9}+1\right) }{ \log n}} \\ &=&\frac{2+\frac{\log 1}{\displaystyle\lim_{n\rightarrow \infty }\log n}}{% \frac{10}{3}+\frac{1}{3}\frac{\log 1}{\displaystyle\lim_{n\rightarrow \infty }\log n}} \\ &=&\frac{2+0}{\frac{10}{3}+\frac{1}{3}\times 0}=\frac{3}{5}. \end{eqnarray*}

0

Solution with help of L'Hôpital's rule.

L'Hôpital's rule tells us, that if we have real valued functions $f$ and $g$ and assumed to be differentiable on an open interval with endpoint $a$, and additionally $g'(x)=0$ on the interval. It is also assumed that $\lim_{x\to\infty}\frac{f'(x)}{g'(x)}=L$ (Let $a$ and $L$ be extended real numbers) If $\lim_{x\to\infty}f(x)=\lim_{x\to\infty}g(x)=\infty $ , then $\lim_{x\to\infty}\frac{f(x)}{g(x)}=L$

I'll leave you check, that all condition are satisfied for your two function,

$f(n)=\log (1 - n + n^2)$ and, $g(n)=\log (1 + n + n^{10})^{1/3}=\frac{1}{3}\log (1 + n + n^{10}).$

Now, let us find the derivatives: $f'(n)$ and $g'(n)$:

$f'(n)=\frac{(1 - n + n^2)'}{1 - n + n^2}=\frac{2n-1}{1 - n + n^2}$

$g'(n)=\frac{1}{3}\frac{(1 + n + n^{10})'}{1 + n + n^{10}}=\frac{1}{3}\frac{(1 + 10n^{9})}{1 + n + n^{10}}$

Now,

$\frac{f'(n)}{g'(n)}=\frac{\frac{2n-1}{1 - n + n^2}}{\frac{1}{3}\frac{(1 + 10n^{9})}{1 + n + n^{10}}}=3\frac{(2n-1)(1 + n + n^{10})}{(1 - n + n^2)(1 + 10n^{9})}=\frac{2n^{11}+...}{10n^{11}+...}$

The limit of last expression is,

$\lim_{n\to\infty}\frac{f'(n)}{g'(n)}=\lim_{n\to\infty}3\frac{2n^{11}+...}{10n^{11}+...}=3\frac{2}{10}=\frac{3}{5}$

Thus,

$\lim_{n\to\infty}\frac{f(n)}{g(n)}=\frac{3}{5}$